Which of the following combinations of side lengths would NOT form a triangle with vertices X, Y, and Z?
A.
XY = 7 mm , YZ = 14 mm , XZ = 25 mm
B.
XY = 11 mm , YZ = 18 mm , XZ = 21 mm
C.
XY = 11 mm , YZ = 14 mm , XZ = 21 mm
D.
XY = 7 mm , YZ = 14 mm , XZ = 17 mm

Answers

Answer 1

The combinations of side lengths that would NOT form a triangle with vertices X, Y, and Z is 7 mm , YZ = 14 mm , XZ = 25 mm.

option A.

What are the possible lengths of triangle?

The lengths of triangle are determined base a given set of rules;

let a, b, and c be the side lengths of a triangle;

Based on the rules of side lengths of triangles, the sum of length a and b must be greater than c, or the sum of a and c must be greater than b or the sum of b and c must be greater than a.

For option A;

7 mm + 14 mm < 25 mm (this cannot be)

For option B;

11 mm + 18 mm > 21 mm (this will work)

For option C;

11 mm + 14 mm > 21 mm (this will work)

For option D;

7 mm + 14 mm > 17 mm (this will work)

Learn more about lengths of triangle here: https://brainly.com/question/24597672

#SPJ1


Related Questions

(12.7)
2. A swimming pool is in the shape of a rectangular
prism with a horizontal cross-section 10 feet by 20
feet. The pool is 5 feet deep and filled to capacity.
Water has a density of approximately 60 pounds
per cubic foot
What is the approximate mass of water in the pool?
A. 8,000 lb.
B.
12,500 lb.
C
16,700 lb.
D. 60,000 lb.

Answers

Answer:

Step-by-step explanation:

The volume of the pool can be calculated as:

Volume = length x width x height

Volume = 10 ft x 20 ft x 5 ft

Volume = 1000 cubic feet

The mass of the water in the pool can be calculated as:

Mass = Volume x Density

Mass = 1000 cubic feet x 60 pounds/cubic foot

Mass = 60,000 pounds

Therefore, the approximate mass of water in the pool is 60,000 lb , which corresponds to option D.

At sunrise donuts you can buy 6 donuts and 2 kolaches for $8.84. On koalches and 4 donuts would cost $5.36. What is the price of one donut at Sunrise Donuts?

Answers

Let x be the price of one donut and y be the price of one kolache. Then we have:

6x + 2y = 8.84 4x + y = 5.36

We can solve for y by multiplying the second equation by -2 and adding it to the first equation:

6x + 2y = 8.84 -8x - 2y = -10.72

-2x = -1.88

Dividing both sides by -2, we get:

x = 0.94

This means that one donut costs $0.94

A circle is circumscribed around a regular octagon with side lemgths of 10 feet. Another circle is inscribed inside the octagon. Find the area. Of the ring created by the two circles. Round the respective radii of the circles to two decimals before calculating the area

Answers

The area of the ring is 1,462.81 square feet, under the condition that a circle is circumscribed around a regular octagon with side lengths of 10 feet.

The area of the ring formed by the two circles can be evaluated using the formula for the area of a ring which is

Area of ring = π(R² - r²)

Here
R = radius of the larger circle
r = smaller circle radius

The radius of the larger circle is equal to half the diagonal of the octagon which is 10 feet. Applying Pythagoras theorem, we can evaluate that the length of one side of the octagon is 10/√2 feet.
Radius of the larger circle is

R = 5(10/√2)
= 25√2/2 feet
≈ 17.68 feet

Staging these values into the formula for the area of a ring,

Area of ring = π(17.68² - 10²) square feet

Area of ring ≈ 1,462.81 square feet
To learn more about Pythagoras theorem theoretheore
https://brainly.com/question/343682
#SPJ4

Find the volume of the solid generated when the right triangle below is rotated about
side IK. Round your answer to the nearest tenth if necessary.

Answers

The volume of the solid generated when the right triangle below is rotated about side IK is: 37.7 units²

What is the volume of a cone?

The three-dimensional figure that is formed by rotating a triangle about it's height is called a Cone.

Where:

The triangle base length will be seen to become the radius of the cone

The triangle height will be seen to become the height of the cone

The formula for the volume of a cone is expressed as:

V = ¹/₃πr²h

Where:

r refers to the radius

h refers to the height

Therefore, we can say that the volume will be expressed as:

V = ¹/₃ * π * 2² * 9

V = 37.7 units²

Thus, that is the  volume of the solid generated when the right triangle below is rotated about side IK.

Read more about Volume of Cone at: https://brainly.com/question/1082469

#SPJ1

Based on the following calculator output, determine the mean of the dataset, rounding to the nearest 100th if necessary.


1-Var-Stats


1-Var-Stats


x


Ë


=


265. 857142857


x


Ë


=265. 857142857


Σ


x


=


1861


Σx=1861


Σ


x


2


=


510909


Σx


2


=510909


S


x


=


51. 8794389954


Sx=51. 8794389954


Ï


x


=


48. 0310273869


Ïx=48. 0310273869


n


=


7


n=7


minX


=


209


minX=209


Q


1


=


221


Q


1


â


=221


Med


=


252


Med=252


Q


3


=


311


Q


3


â


=311


maxX


=


337


maxX=337

Answers

The mean of the dataset, rounded to the nearest hundredth, is approximately 265.86.

Calculate the mean of the dataset from calculator?

The mean, also known as the average, is a measure of central tendency that represents the typical value of a dataset. It is calculated by summing up all the values in the dataset and dividing the sum by the number of values.

To calculate the mean of the dataset from the calculator output, we need to use the following formula:

mean = Σx / n

where Σx is the sum of all the values in the dataset, and n is the number of values in the dataset.

From the calculator output, we can see that:

Σx = 1861

n = 7

Substituting these values into the formula, we get:

mean = 1861 / 7

mean = 265.857142857

However, the problem asks us to round the mean to the nearest hundredth, so we need to round the answer to two output decimal places. To do this, we look at the third decimal place of the answer, which is 7, and we check the next decimal place, which is 1. Since 1 is less than 5, we leave the third decimal place as it is and drop all the decimal places after it. Therefore, the rounded mean is:

mean  ≈ 265.86

learn more about outputhttps:/

brainly.com/question/31878362

3SPJ11

15
Find the first and second derivatives. y = - 5x4+1 dy dx 승 라 ||| dx2

Answers

The first derivative is dy/dx = [tex]-20x^3[/tex], and the second derivative is d^2y/dx^2 = -60x^2.

The function you provided is: y =[tex]-5x^4 + 1[/tex]

To find the first derivative (dy/dx), we'll use the power rule which states that if y = x^n, then dy/dx =[tex]n * x^(n-1)[/tex].

Applying this rule to each term, we get: dy/dx = [tex]d(-5x^4)/dx + d(1)/dx[/tex]dy/dx =[tex]-5(4x^(4-1)) + 0[/tex] (since the derivative of a constant is 0) dy/dx = [tex]-20x^3[/tex]

Now, to find the second derivative [tex](d^2y/dx^2)[/tex], we'll differentiate the first derivative again using the power rule: [tex]d^2y/dx^2 = d(-20x^3)/dx d^2y/dx^2 = -20(3x^(3-1)) d^2y/dx^2 = -60x^2[/tex]

So, the first derivative is dy/dx = [tex]-20x^3[/tex], and the second derivative is [tex]d^2y/dx^2 = -60x^2.[/tex]

Learn more about power rule,

https://brainly.com/question/29288036

#SPJ11

What is question asking??? All I need is one example and I get the rest I just don’t understand the assignment

Answers

It's actually asking you to find the angles within the circles and match it with the angles it's supposed to be. For example if angle COE is 90° (im taking a fake angle value), you should put arc CE <——> 90

If you are still confused and need me to do it so that you can understand what I mean, reply and I'll help you!

Answer:

See below

Step-by-step explanation:

The objective of the question has been well explained by user vaishub1101.

I am just adding an additional hint and one answer to get you going

Since you just needed one example, I am providing just that
One thing to note in the figure is that segments DEF, ACD and ABF are all tangents to the circle. This fact is important since at the point of tangency (where the tangent touches the circle), the tangent to a circle is always perpendicular to the radius.

Using this knowledge and the given angles we can compute all the other angles but not the arc length [tex]\frown \atop {CE}[/tex] since to find arc length we need the value of the radius

As an example to help you get going,
[tex]\angle{DFA} \longleftrightarrow 58^\circ[/tex]

You would drag the tile with ∠DFA to the top left box and the tile with 58° to the top right box

I am sure you can figure out the rest or else user vaishub1101 can help you out with the rest

Explain how to find the measure of angles a and b has a measure of 36 degrees

Answers

The measure of angles a and b is 36 degrees if they are alternate interior angles formed by a transversal intersecting two parallel lines.

How to find the measure of angles a and b with a measure of 36 degrees?

To find the measure of angles a and b when angle b has a measure of 36 degrees, we need additional information.

If we assume that angles a and b are adjacent angles formed by two intersecting lines, then we can use the fact that adjacent angles are supplementary, meaning their measures add up to 180 degrees. Since angle b has a measure of 36 degrees, we subtract it from 180 to find angle a.

Thus, angle a = 180 - 36 = 144 degrees. Therefore, angle a has a measure of 144 degrees when angle b has a measure of 36 degrees.

Learn more about measure

brainly.com/question/4725561
#SPJ11

Find the value of each variable. For theâ circle, the dot represents the center.



A four sided polygon is inside a circle such that each vertex of the polygon is a point on the circle. The top and bottom sides of the polygon slowly rise from left to right. The left and right sides of the polygon quickly fall from left to right. The angle measures of the polygon are as follows, clockwise from the top left: "c" degrees, 123 degrees, 92 degrees, and "d" degrees. The arc bounded by the left side of the polygon is labeled 94 degrees. The arc bounded by the right side of the polygon is labeled "b" degrees. The arc bounded by the bottom side of the polygon is labeled "a" degrees.



123 degrees



92 degrees



94 degrees



c degrees



d degrees



b degrees



a degrees

Answers

The values of the variables are:

c = 86 degrees

d = 168 degrees

a = 57 degrees

b = 94 degrees

Since the polygon is inscribed in a circle, the opposite angles of the polygon are supplementary. Thus, we have:

The top and bottom angles of the polygon are supplementary to angle "d":

c + 92 + 123 = 180 + d

The left and right angles of the polygon are supplementary to angle "c":

c + 94 = 180, so c = 86

The angle "a" is supplementary to angle "d":

a + 123 = 180 + d

The angle "b" is supplementary to angle "c":

b + 86 = 180

Substituting the values of "c" and solving the system of equations, we get:

d = 168

a = 57

b = 94

Therefore, the values of the variables are:

c = 86 degrees

d = 168 degrees

a = 57 degrees

b = 94 degrees

To know more about the polygon

https://brainly.com/question/24464711

#SPJ4

You have $20 to spend. You go to the store and buy a bouncy ball for an unknown amount of money and then you buy a glider airplane for $3. If you have $15 left over, how much did you spend on the bouncy ball?

Answers

Step-by-step explanation:

$20-$3-$15= $2

the amount of money spent on the bouncy ball is $2

find the exact value of z.

Answers

when running a line, in a right-triangle, from the 90° angle perpendicular to its opposite side, we will end up with three similar triangles, one Small, one Medium and a containing Large one.

Check the picture below.

Let f(x) = Show that there is no value c E (1,4) such that f'(c) = f(4) – f(1)/4-1. Why is this not a contradiction of the Mean Value Theorem?

Answers

Derivative f'(c) equals the average rate of change of f(x) over the interval [1, 4], which is given by (f(4) - f(1))/(4 - 1).

It's not a contradiction of the Mean Value Theorem, as we don't have sufficient information to confirm if the conditions for applying the MVT are met.

A more detailed explanation of the answer.

We need to discuss the Mean Value Theorem and determine if it's a contradiction for the given function.

Let f(x) be a continuous function on the interval [1, 4] and differentiable on the open interval (1, 4). According to the Mean Value Theorem (MVT), if these conditions are met, there exists a value c in the open interval (1, 4) such that the derivative f'(c) equals the average rate of change of f(x) over the interval [1, 4], which is given by (f(4) - f(1))/(4 - 1).

However, in your question, the function f(x) is not specified. We cannot determine whether f(x) is continuous on [1, 4] and differentiable on (1, 4) without knowing its specific form. Therefore, we cannot conclude that the MVT is applicable in this case.

So, it's not a contradiction of the Mean Value Theorem, as we don't have sufficient information to confirm if the conditions for applying the MVT are met. If you could provide the specific function f(x), we could further analyze the situation and determine if the MVT can be applied.

Learn more about Mean Value Theorem.

brainly.com/question/29107557

#SPJ11

An investment of $4000 is deposited into an account in which interest is compounded continuously. complete the table by filling in the amounts to which the investment grows at the indicated interest rates. (round your answers to the nearest cent.)
t = 4 years

Answers

The investment grows to $4,493.29 at 2% interest, $4,558.56 at 3% interest, $4,625.05 at 4% interest, $4,692.79 at 5% interest, and $4,761.81 at 6% interest after 4 years of continuous compounding.

To solve this problem, we need to use the formula for continuous compound interest:
A = Pe^(rt)

Where A is the amount after t years, P is the initial principal, e is the mathematical constant approximately equal to 2.71828, r is the annual interest rate, and t is the time in years.

Using the given information, we can fill in the table as follows:

Interest Rate | Amount after 4 years
--------------|---------------------
2%            | $4,493.29
3%            | $4,558.56
4%            | $4,625.05
5%            | $4,692.79
6%            | $4,761.81

To find the amount after 4 years at each interest rate, we plug in the values of P, r, and t into the formula and simplify:

2%: A = $4000 * e^(0.02*4) = $4,493.29
3%: A = $4000 * e^(0.03*4) = $4,558.56
4%: A = $4000 * e^(0.04*4) = $4,625.05
5%: A = $4000 * e^(0.05*4) = $4,692.79
6%: A = $4000 * e^(0.06*4) = $4,761.81

Therefore, the investment grows to $4,493.29 at 2% interest, $4,558.56 at 3% interest, $4,625.05 at 4% interest, $4,692.79 at 5% interest, and $4,761.81 at 6% interest after 4 years of continuous compounding.

Know more about interest here:

https://brainly.com/question/25720319

#SPJ11

On March 1 a commodity's spot price is $60 and its August futures price is $59. On July 1 the spot price is $64 and the

August futures price is $63. 50. A company entered into futures contracts on March 1 to hedge its purchase of the

commodity on July 1. It closed out its position on July 1. What is the effective price (after taking account of hedging) paid

by the company?

Answers

The effective price paid by the company after taking account of hedging would be $63.50, which is the August futures price on July 1. Calculate the profit or loss on the futures contracts and subtract that from the spot price on July 1, to determine the effective.

By entering into futures contracts on March 1, the company was able to lock in the price of $59 for the commodity, when the spot price was $60 and the futures price was $59, the difference between the futures price and the spot price on March 1 was $1 ($60 - $59), so the company had to pay an extra $1 per unit to hedge its purchase.

When the spot price increased to $64 on July 1, the company was still able to purchase the commodity at the lower hedged price of $59, plus the cost of the futures contract, which resulted in an effective price of $63.50. Overall, hedging helped the company mitigate the risk of price volatility and ensured a more predictable cost for the commodity purchase.

Effective price = Spot price - Profit from futures contracts

Effective price = $64 - $0.50(The difference between the futures price and the spot price on July 1 was $0.50 ($64 - $63.50))

Effective price = $63.50 per unit

Therefore, the effective price paid by the company after taking into account hedging was $63.50 per unit.

To learn more about purchase: https://brainly.com/question/27975123

#SPJ11

A Film crew is filming an action movie where a helicopter needs to pick up a stunt actor located on the side of a canyon actor is 20 feet below the ledge of the canyon the helicopter is 30 feet above the canyon. Which of the following expressions represents the length of rope that needs to be lowered from the helicopter to reach the stunt actor

Answers

The expression that represents the length of rope that needs to be lowered is 30 - -20

Which expression represents the length of rope that needs to be lowered

From the question, we have the following parameters that can be used in our computation:

canyon actor is 20 feet below the ledge of the canyon Helicopter is 30 feet above the canyon

Using the above as a guide, we have the following:

Length of rope = helicopter - canyon

So, we have

Length of rope = 30 - -20

Evaluate

Length of rope = 50

Hence, the length of rope is 50 feet

Read more about lengths at

https://brainly.com/question/24487155

#SPJ1

Can please write answer in box Please Thank you
Find the total differential. w = x15yz11 + sin(yz) = dw =

Answers

The total differential of w is given by dw = (∂w/∂x)dx + (∂w/∂y)dy + (∂w/∂z)dz + (∂w/∂z)(∂z/∂y)dy + (∂w/∂z)(∂z/∂z)dz.

Differentiation is a process of finding the changes in any function with a small change in By differentiation, it can be checked that how much a function changes and it also shows the way of change Differentiation is being used cost, production and other management decisions. It gives the rate of change independent variable with respect to the independent variable.                                                                                                             First, let's get the partial derivatives of w with respect to x, y, and z: ∂w/∂x = 15x^14yz^11, ∂w/∂y = x^15z^11cos(yz), ∂w/∂z = 11x^15y^z^10 + x^15y^11cos(yz). Next, we need to find (∂w/∂z)(∂z/∂y): ∂z/∂y = cos(y)
So, (∂w/∂z)(∂z/∂y) = x^15y^11z^10cos(y). Substituting these values into the formula for the total differential, we get: dw = (15x^14yz^11)dx + (x^15z^11cos(yz))dy + (11x^15y^z^10 + x^15y^11cos(yz))dz + (x^15y^11z^10cos(y))dy
Simplifying, we get: dw = 15x^14yz^11dx + x^15z^11cos(yz)dy + (11x^15y^z^10 + x^15y^11cos(yz) + x^15y^11z^10cos(y))dz.

Learn more about total differential here, https://brainly.com/question/28099315

#SPJ11

1. If a 20 inch pizza costs $13, how many square inches of pizza do you
for 1 dollar? In other words, what is the unit rate per one dollar?

Answers

Answer:

I think you get 0.65 inches of pizza for 1 dollar

Step-by-step explanation:

$13 divided by 20 inches = 0.65

If the cost and revenue functions (in dollars) for producing x washing machines is given by C(x) = 10,000+ 0.7x² and R(x) =0.3x² , find the number of washing machines to produce that will maximize profit. You must use Calculus methods to receive credit

Answers

Producing 0 washing machines is not a practical solution for a company.

To maximize profit, we need to find the difference between revenue and cost functions, which gives us the profit function P(x):

P(x) = R(x) - C(x) = (0.3x²) - (10,000 + 0.7x²)

Simplify the profit function:

P(x) = -0.4x² + 10,000

Now, to maximize profit, we'll find the critical points by taking the first derivative of P(x) with respect to x:

P'(x) = dP(x)/dx = -0.8x

Set P'(x) to zero and solve for x:

-0.8x = 0
x = 0

Since the profit function P(x) is a quadratic with a negative leading coefficient, the maximum value will occur at the critical point x = 0. However, producing 0 washing machines is not a practical solution for a company.

To maximize profit while producing washing machines, the company should consider other factors beyond the given cost and revenue functions, such as market demand and production capacity.

To learn more about critical points, refer below:

https://brainly.com/question/29144288

#SPJ11

Make 20 questions subtraction and addition of fractions only like this

e. G saina ,anika and mercy got some money saina got 1/6 and mercy got 3/4 and anka got the rest

a. Caculate the fraction of saina and mercy

b. What is anika s share in fractions

c. How much was the money

you can use money,fruits or any thing

due 24/04/22

Answers

20 questions subtraction and addition of fractions only like this:

A recipe calls for 2/3 cup of flour and 1/4 cup of sugar. What is the total amount of flour and sugar needed in fraction?

John had 3/4 of a pizza and gave 1/3 of it to his friend. What fraction of the pizza does John have left?If 2/5 of a cake is chocolate and 1/5 is vanilla, what fraction of the cake is another flavor?If Tom can run 5/6 of a mile in 4 minutes, how many minutes will it take him to run 1 mile?If 3/8 of a bag of apples is rotten, what fraction of the bag is not rotten?A store had 5/6 of its shelves stocked with books. If 1/4 of the books were science books, what fraction of the shelves were stocked with science books?If Maria has 3/8 of a tank of gas and she uses 1/4 of it to drive to work, what fraction of the tank is left when she arrives at work?A recipe calls for 1/3 cup of sugar and 1/4 cup of butter. What is the total amount of sugar and butter needed in fraction?If 2/3 of a box of crayons is blue and 1/4 of the box is red, what fraction of the box is another color?If a recipe calls for 3/4 cup of milk and you have only 1/2 cup, what fraction of milk do you need to buy to have enough?If a school has 7/8 of its students enrolled in math and 3/4 of those enrolled in math are also enrolled in science, what fraction of the school is enrolled in both math and science?A store had 1/2 of its apples on sale for 1/4 off. What fraction of the original price did a customer pay for a discounted apple?If a train travels 1/2 of a mile in 2 minutes, how long will it take to travel 1 mile?A recipe calls for 2/3 cup of flour and 1/4 cup of sugar. What is the total amount of flour and sugar needed in fraction?If a recipe calls for 3/4 cup of oil and you have only 1/3 cup, what fraction of oil do you need to buy to have enough?If 3/8 of a class is girls and 2/5 of the girls have brown hair, what fraction of the class is girls with brown hair?If a recipe calls for 1/2 cup of brown sugar and 1/4 cup of white sugar, what is the total amount of sugar needed in fraction?If a store sells 3/4 of a bag of apples and has 2/3 of a bag left, what fraction of the original bag is left?If a car travels 3/4 of a mile in 2 minutes, how long will it take to travel 1 mile?If a recipe calls for 1/3 cup of butter and you have only 1/6 cup, what fraction of butter do you need to buy to have enough?If a recipe calls for 1/4 cup of honey and 1/3 cup of sugar, what is the total amount of honey and sugar needed in fraction?

To know more about fractions, refer to the link below:

https://brainly.com/question/29092269#

#SPJ11

Gazza and Julia have each cut a rectangle out of paper. One side is 10 cm. The other side is n cm. (a) They write down expressions for the perimeter of the rectangle. Julia writes Gazza writes 2n+20 2(n + 10) Put a circle around the correct statement below.
Julia is correct and Gazza is wrong.
Gazza is correct and julia is wrong.
Both are correct.
Both are wrong.​

Answers

The correct statement regarding the perimeter of the rectangle is given as follows:

Both are correct.

What is the perimeter of a polygon?

The perimeter of a polygon is given by the sum of all the lengths of the outer edges of the figure, that is, we must find the length of all the edges of the polygon, and then add these lengths to obtain the perimeter.

The rectangle in this problem has:

Two sides of n cm.Two sides of 10 cm.

Hence the perimeter is given as follows:

2 x 10 + 2 x n = 2 x (10 + n) = 20 + 2n = 2n + 20 cm.

Hence both are correct.

More can be learned about the perimeter of a polygon at https://brainly.com/question/3310006

#SPJ1

Help Mr. Johnson has a swimming pool in the shape of a rectangular prism. The dimensions of the pool are 2. 5 meters by 4. 5 meters. He fills the pool with 1. 2 meters of water. What is the volume of water in Mr. Johnson's pool?​

Answers

There are 13.5 cubic meters of water in Mr. Johnson's pool.

Volume is defined as the space occupied within the boundaries of an object in three-dimensional space.

To find the volume of water in Mr. Johnson's pool, we need to multiply the length, width, and depth of the pool. The length is 2.5 meters, the width is 4.5 meters, and the depth of water is 1.2 meters.

So, the volume of water in Mr. Johnson's pool is:

2.5 meters x 4.5 meters x 1.2 meters = 13.5 cubic meters

Therefore, there are 13.5 cubic meters of water in Mr. Johnson's pool.

Learn more about volume,

https://brainly.com/question/27710307

#SPJ11

Mr. Smith invested $2500 in a savings account that earns 3% interest compounded


annually. Find the following:


1. Is this exponential growth or exponential decay?


2. Domain


3. Range


4. Y-intercept


5. Function Rule

Answers

The 99% confidence interval for the population mean is between 39.18 and 62.82, assuming that the population is normally distributed.

How to find the range of the population?

To construct a confidence interval for the population mean, we need to make certain assumptions about the distribution of the sample data and the population. In this case, we assume that the population is normally distributed, the sample size is small (less than 30), and the standard deviation of the population is unknown but can be estimated from the sample data.

Using these assumptions, we can calculate the confidence interval as:

CI = X ± tα/2 * (s/√n)

Where X is the sample mean, tα/2 is the critical value of the t-distribution with degrees of freedom (n-1) and a confidence level of 99%, s is the sample standard deviation, and n is the sample size.

Plugging in the values from the provided data, we get:

CI = 51 ± 2.898 * (17/√18)

CI = (39.18, 62.82)

Therefore, with 99% confidence, we can estimate that the population mean is between 39.18 and 62.82 based on the provided data.

Learn more about  intervals

brainly.com/question/30486507

#SPJ11

let s be a set. suppose that relation r on s is both symmetric and antisymmetric. prove that r ⊆rdiagonal

Answers

We have shown that if r is both symmetric and antisymmetric, then r is a subset of the diagonal relation on s, i.e., r ⊆ diagonal.

If the relation r on s is both symmetric and antisymmetric, then for any elements a and b in s, we have:

If (a, b) is in r, then (b, a) must also be in r because r is symmetric.

If (a, b) and (b, a) are both in r, then a = b because r is antisymmetric.

Now, we want to show that r is a subset of the diagonal relation on s, which is defined as:

diagonal = {(a, a) | a ∈ s}

To prove this, we need to show that for any pair (a, b) in r, (a, b) must also be in the diagonal relation. Since r is a relation on s, (a, b) ∈ s × s, which means that both a and b are elements of s.

Since (a, b) is in r, we know that (b, a) must also be in r, by the symmetry of r. Therefore, we have:

(a, b) ∈ r and (b, a) ∈ r

By the antisymmetry of r, this implies that a = b. Therefore, (a, b) is of the form (a, a), which is an element of the diagonal relation.

Therefore, we have shown that if r is both symmetric and antisymmetric, then r is a subset of the diagonal relation on s, i.e., r ⊆ diagonal.

Learn more about antisymmetric

https://brainly.com/question/26504571

#SPJ4

18. Mr. Kamau wishes to buy some items for his son and daughter. The son's item costs sh. 324 while
the daughter item costs sh. 220 each. Mr. Kamau would like to give each of them equal amount of
money.
a) How many items will each person buys.

Answers

Answer:

if Mr. Kamau wants to give each of his children an equal amount of money, he can either:

Buy 1 item for his son (costing sh. 324) and 0 items for his daughter, giving each child sh. 162.

Buy 1 item for his son (costing sh. 324) and 1 item for his daughter (costing sh. 220), giving each child sh. 272.

Step-by-step explanation:

Let x be the number of daughter items that Mr. Kamau will buy for his daughter. Since the son's item costs sh. 324, we know that each child should receive sh. (324 + 220x)/2.

We want to find how many items each child will buy, so we need to solve for x in the equation:

(324 + 220x)/2 = 220

Multiplying both sides by 2, we get:

324 + 220x = 440

Subtracting 324 from both sides, we get:

220x = 116

Dividing both sides by 220, we get:

x = 0.527

Since we can't buy a fraction of an item, Mr. Kamau should buy either 0 or 1 daughter item for his daughter. If he buys 0 daughter items, he can give his son sh. (324 + 2200)/2 = sh. 162. If he buys 1 daughter item, he can give each child sh. (324 + 2201)/2 = sh. 272. Therefore, the possible scenarios are:

Mr. Kamau buys 0 daughter items. His son buys 1 item and his daughter buys 0 items.

Mr. Kamau buys 1 daughter item. His son buys 1 item and his daughter buys 1 item.

In ΔLMN, m = 59 inches, n = 35 inches and ∠L=82°. Find ∠N, to the nearest degree

Answers

The answer is: ∠N ≈ 33°

To find ∠N in ΔLMN, we can use the Law of Cosines which states that c² = a² + b² - 2abcos(C), where c is the side opposite angle C.

In this case, side LM (m) is opposite angle ∠N, side LN (n) is opposite angle ∠L, and side MN (x) is opposite the unknown angle.

So, we can write:
m² = n² + x² - 2nxcos(82°)

Substituting the given values:
x² = 35² + 59² - 2(35)(59)cos(82°)

Solving for x, we get:
x ≈ 64.27

Now, using the Law of Sines which states that a/sin(A) = b/sin(B) = c/sin(C), we can find ∠N:

sin(∠N)/35 = sin(82°)/64.27

sin(∠N) ≈ 0.5392

∠N ≈ sin⁻¹(0.857) ≈ 32.6344°

Therefore, ∠N ≈ 33° to the nearest degree.

Learn more about: Trigonometry - https://brainly.com/question/31882040

#SPJ11

PLEASE HELP, I NEED IT! AND NO ABSURD ANSWERS! I'll GIVE BRAINLIEST!


The ages of customers at a store are normally distributed with a mean of 45 years and a standard deviation of 13. 8 years.


(a)What is the z-score for a customer that just turned 25 years old? Round to the nearest hundredth.


(b)Give an example of a customer age with a corresponding z-score greater than 2. Justify your answer

Answers

The z-score of the customer that just turned 25 years old is  -1.45. The z-score for an age of 75 years is approximately 2.17, which is greater than 2,  Since a z-score greater than 2 represents a considerable deviation.

(a)

To find the z-score for a customer that just turned 25 years old :

z-score = (x - mean) / standard deviation

Plugging in the values, we get:
z-score = (25 - 45) / 13.8 = -1.45, where x = 25 years, mean  = 45 years, and standard deviation = 13.8 years.
Rounding to the nearest hundredth, the z-score is -1.45.

(b)

To find an example of a customer age with a z-score greater than 2, we need to identify an age that deviates significantly from the mean given the standard deviation. Since a z-score greater than 2 represents a considerable deviation, let's consider an age of 75 years.

Using the same formula as before:

z = (x - μ) / σ

where:

   x is the customer's age (75 years),

   μ is the mean of the distribution (45 years),

   σ is the standard deviation of the distribution (13.8 years).

Calculating the z-score:

z = (75 - 45) / 13.8

z = 2.17

The z-score for an age of 75 years is approximately 2.17, which is greater than 2, fulfilling the requirement of the question.

To learn more about  z-score : https://brainly.com/question/28000192

#SPJ11

Use undetermined coefficients to find the particular solution to
y' +41 -53 = - 580 sin(2t)
Y(t) = ______

Answers

To find the particular solution to this differential equation using undetermined coefficients, we first need to guess the form of the particular solution. Since the right-hand side of the equation is a sinusoidal function, our guess will be a linear combination of sine and cosine functions with the same frequency:

y_p(t) = A sin(2t) + B cos(2t)

We can then find the derivatives of this guess:

y'_p(t) = 2A cos(2t) - 2B sin(2t)
y''_p(t) = -4A sin(2t) - 4B cos(2t)

Substituting these into the differential equation, we get:

(-4A sin(2t) - 4B cos(2t)) + 41(2A cos(2t) - 2B sin(2t)) - 53(A sin(2t) + B cos(2t)) = -580 sin(2t)

Simplifying and collecting terms, we get:

(-53A + 82B) cos(2t) + (82A + 53B) sin(2t) = -580 sin(2t)

Since the left-hand side and right-hand side of this equation must be equal for all values of t, we can equate the coefficients of each trigonometric function separately:

-53A + 82B = 0
82A + 53B = -580

Solving these equations simultaneously, we get:

A = -23
B = -15

Therefore, the particular solution to the differential equation is:

y_p(t) = -23 sin(2t) - 15 cos(2t)

Adding this to the complementary solution (which is just a constant, since the characteristic equation has no roots), we get the general solution:

y(t) = C - 23 sin(2t) - 15 cos(2t)

where C is a constant determined by the initial conditions.
To solve the given differential equation using the method of undetermined coefficients, we need to identify the correct form of the particular solution.

Given the differential equation:
y'(t) + 41y(t) - 53 = -580sin(2t)

We can rewrite it as:
y'(t) + 41y(t) = 53 + 580sin(2t)

Now, let's assume the particular solution Y_p(t) has the form:
Y_p(t) = A + Bsin(2t) + Ccos(2t)

To find A, B, and C, we will differentiate Y_p(t) with respect to t and substitute it back into the differential equation.

Differentiating Y_p(t):
Y_p'(t) = 0 + 2Bcos(2t) - 2Csin(2t)

Now, substitute Y_p'(t) and Y_p(t) into the given differential equation:
(2Bcos(2t) - 2Csin(2t)) + 41(A + Bsin(2t) + Ccos(2t)) = 53 + 580sin(2t)

Now we can match the coefficients of the similar terms:
41A = 53 (constant term)
41B = 580 (sin(2t) term)
-41C = 0 (cos(2t) term)

Solving for A, B, and C:
A = 53/41
B = 580/41
C = 0

Therefore, the particular solution is:
Y_p(t) = 53/41 + (580/41)sin(2t)

Learn more about coefficients here: brainly.com/question/28975079

#SPJ11

The base of a triangular prisms has an area of 18 square inches if the height of the prism is 9. 5 inches then what what is the volume of the prism

Answers

The volume of the triangular prism is 171 cubic inches.

To find the volume of a triangular prism, you need to multiply the area of the base by the height of the prism. In this case, the base of the prism has an area of 18 square inches and the height is 9.5 inches. So, the volume of the prism can be calculated as follows:

Volume = Base Area x Height
Volume = 18 sq. in. x 9.5 in.
Volume = 171 cubic inches

Therefore, the volume of the triangular prism is 171 cubic inches.

Learn more about  volume of the triangular prism,

https://brainly.com/question/29663752

#SPJ11

In a poll of students at the football championship, 90% of the students say that football is better than basketball.

Explain why it is not a valid conclusion to say that football is more popular than basketball at school.

Suggest a better method of determining which sport is more popular.

Answers

Answer:

Part (a):  Because the survey was held at basketball championship.

Part (b): The survey among students should held at school or in any common place.

Step-by-step explanation:

gabriela is building wooden box that is 15 in. tall and has a rectangular base that is 18 in by 15 in

Answers

A open box without a top 1260 sq. in. wood will Gabriella use.

Since the top of the box is the same area as the base, calculate the base.

B = length × width

Length of the wooden box = 18 in.

Width of the wooden box = 15 in.

B = 18(15) = 270 in.

Calculate the surface area of the box.

Surface Area = 2(B + wh + hl)

h = 15

w × h = 15(15) = 225

h × l = 15(18) = 270

Surface Area of the wooden box = 2(270 + 225 + 270) = 2(765) = 1,530 sq. inches

Subtract the base from the surface area: 1,530 - 270 = 1,260sq. in.

Learn more about Surface area of box at:

https://brainly.com/question/22998786

#SPJ1

The given question is incomplete, complete question is:

Gabriella is beauty a wooden box with a rectangular base that is 18 in by 15 in and is 15 in tall if she wants a open box without a top how much wood will Gabriella use

Other Questions
20 points for this IF RIGHT ANSWER A small Aeroplane of mass 600kg has an electric motor powered by fuel cells. Fuel cells use hydrogen gas and provide an electric current. When the Aeroplane is working, the energy changes are chemical --> electrical --> kineticelectrical --> chemical -->kineticelectrical --> kinetic --> chemicalkinetic --> chemical --> electrical You are working with a client to help them reestablish positive adaptation to their environment. what resource(s) would likely provide relevant and useful information to help you work with this client and why? Helen Holmes makes pottery by hand in her basement. She has 20 hours available each week to make bowls and vases. A bowl requires 3 hours of labor, and a vase requires 2 hours of labor. It requires 2 pounds of special clay to make a bowl and 5 pounds to produce a vase; she is able to acquire 35 pounds of clay per week. Helen sells her bowls for $50 and her vases for $40. She wants to know how many of each item to make each week to maximize her revenue. a. Formulate an integer programming model for this problem. b. Solve this model by using the computer. Compare this solution with the solution without inte-ger restrictions and indicate whether the rounded-down solution would have been optimal A fisherman uses sonar to find a shoal of fish. A pulse of ultrasound is sent out and the reflection is detected 0. 4 seconds later. How long did it take the sound to travel from the boat to the fish? 10. The time it takes to drain a swimming pool varies inversely with the amount of water beingpumped per hour. If a large pool takes 12 hours to empty at 400 gallons an hour, how manyhours faster can the pool be drained if the pumping rate is increased to 500 gallons per hour?(teal) 1. NOUNS AND NAMESCircle the first letter of each noun in these sentences. Then write these letters in order onthe line below number 10. If your answers are correct, you have spelled out seven firstnames.1. The students found an umbrella near Evanston last night.2. Some opinions will be discussed by reporters at their meeting.3. The jury came to an understanding and an acceptance of the judge's notification.4. We ladies thought the investigation was horrible.5. After the scientist tested the acid, the microscope was sent to the academy.6. When the research is published, interest in the artifacts will increase.7. Stop it immediately!8. Many inexpensive radios were purchased last October and November.9. My dad has redone the entrance.10. Please slice the bread. Y= 1/2x+5X _ Y-4. ?-2. ?0. ?2. ? Calculate the poh of a 3.14x10-5 m hisolution.poh = (round to 2 decimal places) The author says this is equivalent to taking a long shower every day for two-and- a half weeks to This is for precalc can someone help please Which answer identifies the climax of thank you ma'am 2 examples of how changes to railroad helped American industries to grow Use the formula d = rt to find the distance traveled in a car driven at 45 miles per hour for 5 hours. Rebecca used 4.25pt of milk in her baking recipe. How many cups of milk did she use? which two immigration policies has the biden administration tried to change, but its efforts have been blocked by judges, leading to long legal battles? (select all that apply, there are one to four possible correct answers) A spinning disk with a mass of 12 kg and a radius of 2 m has an angular velocity of 3 rad/s. What is the kinetic energy of the disk? According to the general procedure of Experiment A2b, 213 mg of (E)-stilbene (180. 25 g/mol) was reacted with 435 mg of pyridinium bromide perbromide (319. 82 g/mol) to afford 342 mg of meso-stilbene dibromide (340. 05 g/mol) as a white solid. Calculate the percent yield for this reaction. Enter your answer as digits only (no units), using the proper number of significant figures Pregnant women are encouraged to get flu shots so that the maternal ________ can pass through the placenta to the fetus A circle is centered at (4, 7) and has a radius of 5. Which of the following is the equation of this circle?Group of answer choices(x 4)2 + (x + 7)2 = 5(x 4)2 + (x + 7)2 = 25(x + 4)2 + (x 7)2 = 5(x + 4)2 + (x 7)2 = 25